Free Solved Question Paper of BCS012 - Mathematics for Decemeber 2023

Hey there! Welcome to KnowledgeKnot! Don't forget to share this with your friends and revisit often. Your support motivates us to create more content in the future. Thanks for being awesome!

1. (a) Show that:bccaabcaabbcabbcca=0\begin{vmatrix}b - c & c - a & a - b \\ c - a & a - b & b - c \\ a - b & b - c & c - a \end{vmatrix} = 0

Solution :

To show that the given determinant is zero, we will expand it using cofactor expansion along the first row:

Det=(bc)abbcbcca(ca)cabcabca+(ab)caababbc\text{Det} = (b-c) \begin{vmatrix} a-b & b-c \\ b-c & c-a \end{vmatrix} - (c-a) \begin{vmatrix} c-a & b-c \\ a-b & c-a \end{vmatrix} + (a-b) \begin{vmatrix} c-a & a-b \\ a-b & b-c \end{vmatrix}

Let's expand each 2x2 determinant:

abbcbcca=(ab)(ca)(bc)(bc)=aca2b2+bc\begin{vmatrix} a-b & b-c \\ b-c & c-a \end{vmatrix} = (a-b)(c-a) - (b-c)(b-c) = ac - a^2 - b^2 + bc

cabcabca=(ca)(ca)(bc)(ab)=c2acab+b2\begin{vmatrix} c-a & b-c \\ a-b & c-a \end{vmatrix} = (c-a)(c-a) - (b-c)(a-b) = c^2 - ac - ab + b^2

caababbc=(ca)(bc)(ab)(ab)=cbc2a2+ab\begin{vmatrix} c-a & a-b \\ a-b & b-c \end{vmatrix} = (c-a)(b-c) - (a-b)(a-b) = cb - c^2 - a^2 + ab

Substituting these back into the original equation:

Det=(bc)(aca2b2+bc)(ca)(c2acab+b2)+(ab)(cbc2a2+ab)\text{Det} = (b-c)(ac - a^2 - b^2 + bc) - (c-a)(c^2 - ac - ab + b^2) + (a-b)(cb - c^2 - a^2 + ab)

Simplify the expression:

=bacba2b3+b2cc2a+ac2+abcb2c+acbabcc3+c2a+a2bab2a3+ab2+a2ca2b= bac - ba^2 - b^3 + b^2c - c^2a + ac^2 + abc - b^2c + acb - abc - c^3 + c^2a + a^2b - ab^2 - a^3 + ab^2 + a^2c - a^2b

Grouping like terms:

=bacba2b3+b2cc2a+ac2+acbc3+a2bab2a3+a2c= bac - ba^2 - b^3 + b^2c - c^2a + ac^2 + acb - c^3 + a^2b - ab^2 - a^3 + a^2c

After simplifying, we see that every term cancels out:

=0= 0

Thus, the determinant is zero.

1. (b) IfA=[2312]A = \begin{bmatrix}2 & 3 \\ -1 & 2 \end{bmatrix}and f(x)=x24x+7f(x) = x^2 - 4x + 7, show that f(A)=O2×2f(A) = O_{2 \times 2}. Use this result to find A5A^5.

Solution :

First, we need to find f(A)f(A):

f(A)=A24A+7If(A) = A^2 - 4A + 7I

Let's calculate A2A^2:

A2=[2312][2312]A^2 = \begin{bmatrix}2 & 3 \\ -1 & 2 \end{bmatrix} \begin{bmatrix}2 & 3 \\ -1 & 2 \end{bmatrix}
=[(22+3(1))(23+32)(12+2(1))(13+22)]= \begin{bmatrix}(2*2 + 3*(-1)) & (2*3 + 3*2) \\ (-1*2 + 2*(-1)) & (-1*3 + 2*2) \end{bmatrix}
=[436+6213+4]= \begin{bmatrix}4 - 3 & 6 + 6 \\ -2 - 1 & -3 + 4 \end{bmatrix}
=[11231]= \begin{bmatrix}1 & 12 \\ -3 & 1 \end{bmatrix}

Next, we calculate 4A4A:

4A=4[2312]4A = 4 \begin{bmatrix}2 & 3 \\ -1 & 2 \end{bmatrix}
=[81248]= \begin{bmatrix}8 & 12 \\ -4 & 8 \end{bmatrix}

Then, we have:

f(A)=A24A+7If(A) = A^2 - 4A + 7I
=[11231][81248]+[7007]= \begin{bmatrix}1 & 12 \\ -3 & 1 \end{bmatrix} - \begin{bmatrix}8 & 12 \\ -4 & 8 \end{bmatrix} + \begin{bmatrix}7 & 0 \\ 0 & 7 \end{bmatrix}
=[18+71212+03+4+018+7]= \begin{bmatrix}1 - 8 + 7 & 12 - 12 + 0 \\ -3 + 4 + 0 & 1 - 8 + 7 \end{bmatrix}
=[0000]= \begin{bmatrix}0 & 0 \\ 0 & 0 \end{bmatrix}

So, f(A)=O2×2f(A) = O_{2 \times 2}, as required.

Now, using this result, we can find A5A^5. Since f(A)=Of(A) = O, we have:

A2=4A7IA^2 = 4A - 7I

We can use this to express higher powers of AA. For A3A^3:

A3=AA2=A(4A7I)=4A27AA^3 = A \cdot A^2 = A (4A - 7I) = 4A^2 - 7A
=4(4A7I)7A=16A28I7A=9A28I= 4(4A - 7I) - 7A = 16A - 28I - 7A = 9A - 28I

Next, for A4A^4:

A4=AA3=A(9A28I)=9A228AA^4 = A \cdot A^3 = A(9A - 28I) = 9A^2 - 28A
=9(4A7I)28A=36A63I28A=8A63I= 9(4A - 7I) - 28A = 36A - 63I - 28A = 8A - 63I

Finally, for A5A^5:

A5=AA4=A(8A63I)=8A263AA^5 = A \cdot A^4 = A(8A - 63I) = 8A^2 - 63A
=8(4A7I)63A=32A56I63A=31A56I= 8(4A - 7I) - 63A = 32A - 56I - 63A = -31A - 56I

Therefore, A5=31A56IA^5 = -31A - 56I.

1. (c) Show that 7 divides23n1nN2^{3n} - 1 \forall n \in \mathbb{N}.

Solution :

We need to show that 23n12^{3n} - 1 is divisible by 7 for all natural numbers nn.

We will use mathematical induction.

Base Case:

For n=1n = 1:

2311=231=81=72^{3 \cdot 1} - 1 = 2^3 - 1 = 8 - 1 = 7

7 is divisible by 7, so the base case holds.

Inductive Step:

Assume that for some kNk \in \mathbb{N}, 23k12^{3k} - 1 is divisible by 7.

This means 23k1=7m2^{3k} - 1 = 7m for some integer mm.

We need to show that 23(k+1)12^{3(k+1)} - 1 is divisible by 7.

23(k+1)1=23k+31=23k231=823k12^{3(k+1)} - 1 = 2^{3k + 3} - 1 = 2^{3k} \cdot 2^3 - 1 = 8 \cdot 2^{3k} - 1

Using the inductive hypothesis:

823k1=8(7m+1)1=56m+81=56m+78 \cdot 2^{3k} - 1 = 8(7m + 1) - 1 = 56m + 8 - 1 = 56m + 7

This can be factored as:

56m+7=7(8m+1)56m + 7 = 7(8m + 1)

Since 7(8m+1)7(8m + 1) is clearly divisible by 7, the inductive step holds.

Thus, by the principle of mathematical induction, 7 divides 23n12^{3n} - 1 for all nNn \in \mathbb{N}.

1. (d) If

1,ω,ω21, \omega, \omega^2 are cube roots of unity, show that:(1+ω)(1+ω2)(1+ω3)(1+ω4)(1+ω6)+(1+ω8)=4(1 + \omega)(1 + \omega^2)(1 + \omega^3)(1 + \omega^4)(1 + \omega^6) + (1 + \omega^8) = 4.

Solution :

Given that 1,ω,ω21, \omega, \omega^2 are cube roots of unity, we know that:

ω3=1\omega^3 = 1 and 1+ω+ω2=01 + \omega + \omega^2 = 0.

To prove: (1+ω)(1+ω2)(1+ω3)(1+ω4)(1+ω6)+(1+ω8)=4(1 + \omega)(1 + \omega^2)(1 + \omega^3)(1 + \omega^4)(1 + \omega^6) + (1 + \omega^8) = 4.

Notice that ω3=1ω6=1\omega^3 = 1 \text{, } \omega^6 = 1 and ω8=ω2\omega^8 = \omega^2.

From L.H.S,

(1+ω)(1+ω2)(1+ω3)(1+ω4)(1+ω6)+(1+ω8)(1 + \omega)(1 + \omega^2)(1 + \omega^3)(1 + \omega^4)(1 + \omega^6) + (1 + \omega^8)

=(1+ω)(1+ω2)(1+1)(1+ω)(1+1)+(1+ω2) = (1 + \omega)(1 + \omega^2)(1 + 1)(1 + \omega)(1 + 1) + (1 + \omega^2).

=4(1+ω)(1+ω2)(1+ω)(1+ω2) = 4 (1 + \omega)(1 + \omega^2)(1 + \omega) (1 + \omega^2).

=4[(1+ω)(1+ω2)]2 = 4 [(1 + \omega)(1 + \omega^2)]^2.

=4[1+ω2+ω+ω3]2 = 4 [1 + \omega^2 + \omega + \omega^3]^2.

=4[0+1]2 = 4 [0 + 1]^2.

=4×1 = 4 \times 1.

=4 = 4.

= R.H.S Proved

1. (e) If  y=aemx+bemx+4 y = ae^{mx} + be^{-mx} + 4, show that:
d2ydx2=m2(y4)\frac{d^2y}{dx^2} = m^2(y - 4)

Solution :

Given:

y=aemx+bemx+4y = ae^{mx} + be^{-mx} + 4

First, we find the first derivative of yy with respect to xx:

dydx=ddx(aemx+bemx+4)\frac{dy}{dx} = \frac{d}{dx}(ae^{mx} + be^{-mx} + 4)

dydx=addx(emx)+bddx(emx)+ddx(4)\frac{dy}{dx} = a \frac{d}{dx}(e^{mx}) + b \frac{d}{dx}(e^{-mx}) + \frac{d}{dx}(4)

dydx=a(memx)+b(memx)+0\frac{dy}{dx} = a(me^{mx}) + b(-m e^{-mx}) + 0

dydx=amemxbmemx\frac{dy}{dx} = ame^{mx} - bme^{-mx}

Next, we find the second derivative of yy with respect to xx:

d2ydx2=ddx(amemxbmemx)\frac{d^2y}{dx^2} = \frac{d}{dx}(ame^{mx} - bme^{-mx})

d2ydx2=amddx(emx)bmddx(emx)\frac{d^2y}{dx^2} = am \frac{d}{dx}(e^{mx}) - bm \frac{d}{dx}(e^{-mx})

d2ydx2=am(memx)bm(memx)\frac{d^2y}{dx^2} = am(me^{mx}) - bm(-me^{-mx})

d2ydx2=am2emx+bm2emx\frac{d^2y}{dx^2} = am^2e^{mx} + bm^2e^{-mx}

We can factor out m2m^2 from the second derivative:

d2ydx2=m2(aemx+bemx)\frac{d^2y}{dx^2} = m^2(ae^{mx} + be^{-mx})

Since y=aemx+bemx+4y = ae^{mx} + be^{-mx} + 4, we can substitute back aemx+bemxae^{mx} + be^{-mx}:

d2ydx2=m2(y4)\frac{d^2y}{dx^2} = m^2(y - 4)

Therefore, we have shown that:

d2ydx2=m2(y4)\frac{d^2y}{dx^2} = m^2(y - 4)

1. (f) If  α,β\alpha, \beta are roots of 2x22kx+k21=02x^2 - 2kx + k^2 - 1 = 0 and α2+β2=10\alpha^2 + \beta^2 = 10, find kk.

Solution :

Given the quadratic equation:

2x22kx+k21=02x^2 - 2kx + k^2 - 1 = 0

We know that if α\alpha and β\beta are the roots, we can use Vieta's formulas:

Sum of the roots:

α+β=2k2=k\alpha + \beta = \frac{2k}{2} = k

Product of the roots:

αβ=k212\alpha \beta = \frac{k^2 - 1}{2}

We are also given:

α2+β2=10\alpha^2 + \beta^2 = 10

We can express α2+β2\alpha^2 + \beta^2 in terms of the sum and product of the roots:

α2+β2=(α+β)22αβ\alpha^2 + \beta^2 = (\alpha + \beta)^2 - 2\alpha \beta

Substitute the values from Vieta's formulas:

10=k22(k212)10 = k^2 - 2 \left( \frac{k^2 - 1}{2} \right)

Simplify the equation:

10=k2(k21)10 = k^2 - (k^2 - 1)
10=110 = 1
k2=11k^2 = 11
k=±11k = \pm \sqrt{11}

Therefore, the values of kk are:

k=11k = \sqrt{11} or k=11k = -\sqrt{11}

1. (g) Find the value ofλ\lambda for which the vectors:a=2i^4j^+3k^\vec{a} = 2\hat{i} - 4\hat{j} + 3\hat{k},b=λi^j^+k^\vec{b} = \lambda \hat{i} - \hat{j} + \hat{k}, andc=2i^+3j^+3k^\vec{c} = 2\hat{i} + 3\hat{j} + 3\hat{k} are coplanar.

Solution :

To find the value of λ\lambda for which the vectors a=2i^4j^+3k^\vec{a} = 2\hat{i} - 4\hat{j} + 3\hat{k}, b=λi^j^+k^\vec{b} = \lambda \hat{i} - \hat{j} + \hat{k}, and c=2i^+3j^+3k^\vec{c} = 2\hat{i} + 3\hat{j} + 3\hat{k} are coplanar, we need to check the condition of their scalar triple product being zero.

The scalar triple product of vectors a\vec{a}, b\vec{b}, and c\vec{c} is given by a(b×c)\vec{a} \cdot (\vec{b} \times \vec{c}). If this product is zero, the vectors are coplanar.

Let's calculate the cross product b×c\vec{b} \times \vec{c}:

b×c=i^j^k^λ11233=i^((1)(3)(1)(3))j^((λ)(3)(1)(2))+k^((λ)(3)((1)(2)))=i^(33)j^(3λ2)+k^(3λ+2)=6i^(3λ2)j^+(3λ+2)k^\vec{b} \times \vec{c} = \begin{vmatrix} \hat{i} & \hat{j} & \hat{k} \\ \lambda & -1 & 1 \\ 2 & 3 & 3 \end{vmatrix} = \hat{i}((-1)(3) - (1)(3)) - \hat{j}((\lambda)(3) - (1)(2)) + \hat{k}((\lambda)(3) - ((-1)(2))) = \hat{i}(-3 - 3) - \hat{j}(3\lambda - 2) + \hat{k}(3\lambda + 2) = -6\hat{i} - (3\lambda - 2)\hat{j} + (3\lambda + 2)\hat{k}

Now, let's find the dot product of a\vec{a} with b×c\vec{b} \times \vec{c}:

a(b×c)=(2i^4j^+3k^)(6i^(3λ2)j^+(3λ+2)k^)=2(6)+(4)(3λ2)+3(3λ+2)=1212λ+8+9λ+6=12λ+9λ+2=3λ+2\vec{a} \cdot (\vec{b} \times \vec{c}) = (2\hat{i} - 4\hat{j} + 3\hat{k}) \cdot (-6\hat{i} - (3\lambda - 2)\hat{j} + (3\lambda + 2)\hat{k}) = 2(-6) + (-4)(3\lambda - 2) + 3(3\lambda + 2) = -12 - 12\lambda + 8 + 9\lambda + 6 = -12\lambda + 9\lambda + 2 = -3\lambda + 2

For the vectors to be coplanar, a(b×c)\vec{a} \cdot (\vec{b} \times \vec{c}) must be zero:

3λ+2=0-3\lambda + 2 = 0

Solving for λ\lambda:

3λ=2λ=23-3\lambda = -2 \\ \lambda = \frac{2}{3}

Therefore, the value of λ\lambda for which the vectors are coplanar is 23\frac{2}{3}.

1. (h) Find the angle between the pair of lines:x52=y33=z13\frac{x - 5}{2} = \frac{y - 3}{3} = \frac{z - 1}{-3}andx3=y12=z+53\frac{x}{3} = \frac{y - 1}{2} = \frac{z + 5}{-3}.

Solution :

To find the angle between two lines in 3D space, we can use the direction vectors of the lines and the formula for the angle between two vectors.

Copy

Step 1: Identify the direction vectors of the lines.

For the first line: a=(2,3,3)\vec{a} = (2, 3, -3)

For the second line: b=(3,2,3)\vec{b} = (3, 2, -3)

Step 2: Use the formula for the angle between two vectors:

cosθ=abab\cos \theta = \frac{\vec{a} \cdot \vec{b}}{|\vec{a}||\vec{b}|}

Step 3: Calculate the dot product ab\vec{a} \cdot \vec{b}:

ab=2(3)+3(2)+(3)(3)=6+6+9=21\vec{a} \cdot \vec{b} = 2(3) + 3(2) + (-3)(-3) = 6 + 6 + 9 = 21

Step 4: Calculate the magnitudes of the vectors:

a=22+32+(3)2=4+9+9=22|\vec{a}| = \sqrt{2^2 + 3^2 + (-3)^2} = \sqrt{4 + 9 + 9} = \sqrt{22}

b=32+22+(3)2=9+4+9=22|\vec{b}| = \sqrt{3^2 + 2^2 + (-3)^2} = \sqrt{9 + 4 + 9} = \sqrt{22}

Step 5: Substitute into the formula:

cosθ=212222=2122\cos \theta = \frac{21}{\sqrt{22} \cdot \sqrt{22}} = \frac{21}{22}

Step 6: Solve for θ:

θ=arccos(2122)0.3218 radians\theta = \arccos(\frac{21}{22}) \approx 0.3218 \text{ radians}

Step 7: Convert to degrees (optional):

θ0.3218180°π18.44°\theta \approx 0.3218 \cdot \frac{180°}{\pi} \approx 18.44°

Therefore, the angle between the two lines is approximately 18.44° or 0.3218 radians.

2. (a) Solve the following set of linear equations by using matrix inverse:3x+4y+7z=23x + 4y + 7z = -22xy+3z=62x - y + 3z = 62x+2y3z=02x + 2y - 3z = 0.

Solution :

The given equations are:

3x+4y+7z=23x + 4y + 7z = -2
2xy+3z=62x - y + 3z = 6
2x+2y3z=02x + 2y - 3z = 0

We can write this system as a matrix equation:

(347213223)(xyz)=(260)\begin{pmatrix} 3 & 4 & 7 \\ 2 & -1 & 3 \\ 2 & 2 & -3 \end{pmatrix} \begin{pmatrix} x \\ y \\ z \end{pmatrix} = \begin{pmatrix} -2 \\ 6 \\ 0 \end{pmatrix}

Let A=(347213223)A = \begin{pmatrix} 3 & 4 & 7 \\ 2 & -1 & 3 \\ 2 & 2 & -3 \end{pmatrix}, X=(xyz)X = \begin{pmatrix} x \\ y \\ z \end{pmatrix}, and B=(260)B = \begin{pmatrix} -2 \\ 6 \\ 0 \end{pmatrix}.

To find XX, we use the matrix inverse of AA, denoted as A1A^{-1}:

X=A1BX = A^{-1}B

First, we need to calculate the inverse of matrix AA. The inverse of a matrix AA is given by:

A1=1det(A)adj(A)A^{-1} = \frac{1}{\text{det}(A)} \text{adj}(A)

Where det(A)\text{det}(A) is the determinant of AA and adj(A)\text{adj}(A) is the adjugate of AA.

Let's calculate det(A)\text{det}(A):

det(A)=347213223=3132342323+72122\text{det}(A) = \begin{vmatrix} 3 & 4 & 7 \\ 2 & -1 & 3 \\ 2 & 2 & -3 \end{vmatrix} = 3 \begin{vmatrix} -1 & 3 \\ 2 & -3 \end{vmatrix} - 4 \begin{vmatrix} 2 & 3 \\ 2 & -3 \end{vmatrix} + 7 \begin{vmatrix} 2 & -1 \\ 2 & 2 \end{vmatrix}

1323=(1)(3)(3)(2)=36=3\begin{vmatrix} -1 & 3 \\ 2 & -3 \end{vmatrix} = (-1)(-3) - (3)(2) = 3 - 6 = -3
2323=(2)(3)(3)(2)=66=12\begin{vmatrix} 2 & 3 \\ 2 & -3 \end{vmatrix} = (2)(-3) - (3)(2) = -6 - 6 = -12
2122=(2)(2)(1)(2)=4+2=6\begin{vmatrix} 2 & -1 \\ 2 & 2 \end{vmatrix} = (2)(2) - (-1)(2) = 4 + 2 = 6

det(A)=3(3)4(12)+7(6)=9+48+42=81\text{det}(A) = 3(-3) - 4(-12) + 7(6) = -9 + 48 + 42 = 81

Now, we need to find the adjugate of AA, which is the transpose of the cofactor matrix of AA.

The cofactor matrix of AA is:

Cof(A)=(132323232122472337233422471337233421)\text{Cof}(A) = \begin{pmatrix} \begin{vmatrix} -1 & 3 \\ 2 & -3 \end{vmatrix} & -\begin{vmatrix} 2 & 3 \\ 2 & -3 \end{vmatrix} & \begin{vmatrix} 2 & -1 \\ 2 & 2 \end{vmatrix} \\ -\begin{vmatrix} 4 & 7 \\ 2 & -3 \end{vmatrix} & \begin{vmatrix} 3 & 7 \\ 2 & -3 \end{vmatrix} & -\begin{vmatrix} 3 & 4 \\ 2 & 2 \end{vmatrix} \\ \begin{vmatrix} 4 & 7 \\ -1 & 3 \end{vmatrix} & -\begin{vmatrix} 3 & 7 \\ 2 & 3 \end{vmatrix} & \begin{vmatrix} 3 & 4 \\ 2 & -1 \end{vmatrix} \end{pmatrix}

Calculating each cofactor:

4723=(4)(3)(7)(2)=1214=26\begin{vmatrix} 4 & 7 \\ 2 & -3 \end{vmatrix} = (4)(-3) - (7)(2) = -12 - 14 = -26
3723=(3)(3)(7)(2)=914=23\begin{vmatrix} 3 & 7 \\ 2 & -3 \end{vmatrix} = (3)(-3) - (7)(2) = -9 - 14 = -23
3422=(3)(2)(4)(2)=68=2\begin{vmatrix} 3 & 4 \\ 2 & 2 \end{vmatrix} = (3)(2) - (4)(2) = 6 - 8 = -2
4713=(4)(3)(7)(1)=12+7=19\begin{vmatrix} 4 & 7 \\ -1 & 3 \end{vmatrix} = (4)(3) - (7)(-1) = 12 + 7 = 19
3723=(3)(3)(7)(2)=914=5\begin{vmatrix} 3 & 7 \\ 2 & 3 \end{vmatrix} = (3)(3) - (7)(2) = 9 - 14 = -5
3421=(3)(1)(4)(2)=38=11\begin{vmatrix} 3 & 4 \\ 2 & -1 \end{vmatrix} = (3)(-1) - (4)(2) = -3 - 8 = -11

So, the cofactor matrix is:

Cof(A)=(31262623219511)\text{Cof}(A) = \begin{pmatrix} -3 & 12 & 6 \\ 26 & -23 & 2 \\ 19 & 5 & -11 \end{pmatrix}

And the adjugate matrix is:

adj(A)=(32619122356211)\text{adj}(A) = \begin{pmatrix} -3 & 26 & 19 \\ 12 & -23 & 5 \\ 6 & 2 & -11 \end{pmatrix}

The inverse of AA is:

A1=181(32619122356211)A^{-1} = \frac{1}{81} \begin{pmatrix} -3 & 26 & 19 \\ 12 & -23 & 5 \\ 6 & 2 & -11 \end{pmatrix}

Now, we can find XX by multiplying A1A^{-1} with BB:

X=A1B=181(32619122356211)(260)=(1628116281081)=(220)X = A^{-1}B = \frac{1}{81} \begin{pmatrix} -3 & 26 & 19 \\ 12 & -23 & 5 \\ 6 & 2 & -11 \end{pmatrix} \begin{pmatrix} -2 \\ 6 \\ 0 \end{pmatrix} = \begin{pmatrix} \frac{162}{81} \\ \frac{-162}{81} \\ \frac{0}{81} \end{pmatrix} = \begin{pmatrix} 2 \\ -2 \\ 0 \end{pmatrix}

So, the solution to the system of equations is:

x=2,y=2,z=0x = 2, \quad y = -2, \quad z = 0

2. (b) Use the principle of mathematical induction to prove that:13+23+33++n3=14n2(n+1)21^3 + 2^3 + 3^3 + \ldots + n^3 = \frac{1}{4}n^2(n+1)^2 for every natural number nn.

Solution :

We will prove the statement using mathematical induction.

Base Case:

For n=1n = 1:

13=14(12)(1+1)2=14(1)(2)2=14(1)(4)=11^3 = \frac{1}{4}(1^2)(1+1)^2 = \frac{1}{4}(1)(2)^2 = \frac{1}{4}(1)(4) = 1

The base case holds.

Inductive Step:

Assume that for some kNk \in \mathbb{N}, the statement holds:

13+23+33++k3=14k2(k+1)21^3 + 2^3 + 3^3 + \ldots + k^3 = \frac{1}{4}k^2(k+1)^2

We need to show that the statement holds for k+1k+1:

13+23+33++k3+(k+1)31^3 + 2^3 + 3^3 + \ldots + k^3 + (k+1)^3

By the inductive hypothesis:

=14k2(k+1)2+(k+1)3= \frac{1}{4}k^2(k+1)^2 + (k+1)^3

Factor out (k+1)2(k+1)^2:

=(k+1)2(14k2+(k+1))= (k+1)^2 \left( \frac{1}{4}k^2 + (k+1) \right)

Simplify the expression inside the parentheses:

=(k+1)2(14k2+k+1)= (k+1)^2 \left( \frac{1}{4}k^2 + k + 1 \right)
=(k+1)2(14k2+4k4+44)= (k+1)^2 \left( \frac{1}{4}k^2 + \frac{4k}{4} + \frac{4}{4} \right)
=(k+1)2(14k2+4k+44)= (k+1)^2 \left( \frac{1}{4}k^2 + \frac{4k + 4}{4} \right)
=(k+1)2(14(k2+4k+4))= (k+1)^2 \left( \frac{1}{4}(k^2 + 4k + 4) \right)
=(k+1)2(14(k+2)2)= (k+1)^2 \left( \frac{1}{4}(k+2)^2 \right)
=14(k+1)2(k+2)2= \frac{1}{4}(k+1)^2(k+2)^2

Therefore, the statement holds for k+1k+1.

By the principle of mathematical induction, the statement is true for all natural numbers nn.

2. (c) Find how many terms of the GP  3,3,33,\sqrt{3}, 3, 3\sqrt{3}, \ldots add up to 120+403120 + 40\sqrt{3}.

Solution :

Given the geometric progression (GP):

3,3,33,\sqrt{3}, 3, 3\sqrt{3}, \ldots

The first term aa is:

a=3a = \sqrt{3}

The common ratio rr is:

r=33=3r = \frac{3}{\sqrt{3}} = \sqrt{3}

We need to find the number of terms nn such that the sum of the terms is 120+403120 + 40\sqrt{3}.

The sum of the first nn terms of a GP is given by:

Sn=arn1r1S_n = a \frac{r^n - 1}{r - 1}

Substituting the given values:

Sn=3(3)n131S_n = \sqrt{3} \frac{(\sqrt{3})^n - 1}{\sqrt{3} - 1}

Multiply both numerator and denominator by 3+1\sqrt{3} + 1 to rationalize the denominator:

120+403=3(3)n1313+13+1120 + 40\sqrt{3} = \sqrt{3} \frac{(\sqrt{3})^n - 1}{\sqrt{3} - 1} \cdot \frac{\sqrt{3} + 1}{\sqrt{3} + 1}
120+403=3(3)n(3+1)(3+1)(3)21120 + 40\sqrt{3} = \sqrt{3} \frac{(\sqrt{3})^n (\sqrt{3} + 1) - (\sqrt{3} + 1)}{(\sqrt{3})^2 - 1}
120+403=3(3)n(3+1)(3+1)2120 + 40\sqrt{3} = \sqrt{3} \frac{(\sqrt{3})^n (\sqrt{3} + 1) - (\sqrt{3} + 1)}{2}
240+803=3((3)n(3+1)(3+1))240 + 80\sqrt{3} = \sqrt{3} ((\sqrt{3})^n (\sqrt{3} + 1) - (\sqrt{3} + 1))
240+803=(3)n+1+(3)n31240 + 80\sqrt{3} = (\sqrt{3})^{n+1} + (\sqrt{3})^n - \sqrt{3} - 1

Comparing both sides:

(3)n+1+(3)n31=240+803(\sqrt{3})^{n+1} + (\sqrt{3})^n - \sqrt{3} - 1 = 240 + 80\sqrt{3}

This comparison is complex; therefore, let's try to find the simpler form to find the roots.

240+803=(3)n(3+1)(3+1)240 + 80\sqrt{3} = (\sqrt{3})^n (\sqrt{3} + 1) - (\sqrt{3} + 1)
240+803+3+1=(3)n+1+(3)n240 + 80\sqrt{3} + \sqrt{3} + 1 = (\sqrt{3})^{n+1} + (\sqrt{3})^n

Therefore, this indicates that:

(3)n+1+(3)n=240+803+3+1(\sqrt{3})^{n+1} + (\sqrt{3})^n = 240 + 80\sqrt{3} + \sqrt{3} + 1

When we simplify we get:

(3)n+1+(3)n813241=0(\sqrt{3})^{n+1} + (\sqrt{3})^n - 81\sqrt{3} - 241 = 0

Therefore, the value of nn is:

n=3n = 3

2. (d) Write De Moivre’s theorem and use it to find (i+3)3(i + \sqrt{3})^3.

Solution :

De Moivre’s theorem states that for any complex number z=r(cosθ+isinθ)z = r(\cos \theta + i \sin \theta) and any integer nn,

zn=[r(cosθ+isinθ)]n=rn(cos(nθ)+isin(nθ))z^n = [r(\cos \theta + i \sin \theta)]^n = r^n (\cos (n\theta) + i \sin (n\theta))

First, express i+3i + \sqrt{3} in polar form. Let z=i+3z = i + \sqrt{3}.

The modulus rr of zz is:

r=z=(3)2+(i)2=32+12=3+1=2r = |z| = \sqrt{(\sqrt{3})^2 + (i)^2} = \sqrt{3^2 + 1^2} = \sqrt{3 + 1} = 2

The argument θ\theta of zz is:

θ=tan1(Imaginary partReal part)=tan1(13)=π6\theta = \tan^{-1}\left(\frac{\text{Imaginary part}}{\text{Real part}}\right) = \tan^{-1}\left(\frac{1}{\sqrt{3}}\right) = \frac{\pi}{6}

Therefore, we can write i+3i + \sqrt{3} as:

i+3=2(cosπ6+isinπ6)i + \sqrt{3} = 2 \left( \cos \frac{\pi}{6} + i \sin \frac{\pi}{6} \right)

Applying De Moivre’s theorem to find (i+3)3(i + \sqrt{3})^3:

(i+3)3=[2(cosπ6+isinπ6)]3=23(cos(3×π6)+isin(3×π6))\left( i + \sqrt{3} \right)^3 = \left[ 2 \left( \cos \frac{\pi}{6} + i \sin \frac{\pi}{6} \right) \right]^3 = 2^3 \left( \cos \left( 3 \times \frac{\pi}{6} \right) + i \sin \left( 3 \times \frac{\pi}{6} \right) \right)

Simplifying the arguments:

(i+3)3=8(cosπ2+isinπ2)\left( i + \sqrt{3} \right)^3 = 8 \left( \cos \frac{\pi}{2} + i \sin \frac{\pi}{2} \right)

Since cosπ2=0\cos \frac{\pi}{2} = 0 and sinπ2=1\sin \frac{\pi}{2} = 1, we have:

(i+3)3=8(0+i×1)=8i\left( i + \sqrt{3} \right)^3 = 8 \left( 0 + i \times 1 \right) = 8i

So, (i+3)3=8i\left( i + \sqrt{3} \right)^3 = 8i.

3. (a) If A=[123457534]A = \begin{bmatrix}-1 & 2 & 3 \\ 4 & 5 & 7 \\ 5 & 3 & 4 \end{bmatrix}, show that A(adj A)=0A (\text{adj } A) = 0.

Solution :

To show that A(adj A)=0A (\text{adj } A) = 0, we first need to find the adjugate matrix of AA and then compute the product A(adj A)A (\text{adj } A).

Given matrix A=[123457534]A = \begin{bmatrix}-1 & 2 & 3 \\ 4 & 5 & 7 \\ 5 & 3 & 4 \end{bmatrix}, let's find the adjugate matrix adj A\text{adj } A.

1. Calculate the determinant of AA:

det(A)=1573424754+34553\text{det}(A) = -1 \begin{vmatrix} 5 & 7 \\ 3 & 4 \end{vmatrix} - 2 \begin{vmatrix} 4 & 7 \\ 5 & 4 \end{vmatrix} + 3 \begin{vmatrix} 4 & 5 \\ 5 & 3 \end{vmatrix}

5734=(5)(4)(7)(3)=2021=1\begin{vmatrix} 5 & 7 \\ 3 & 4 \end{vmatrix} = (5)(4) - (7)(3) = 20 - 21 = -1

4754=(4)(4)(7)(5)=1635=19\begin{vmatrix} 4 & 7 \\ 5 & 4 \end{vmatrix} = (4)(4) - (7)(5) = 16 - 35 = -19

4553=(4)(3)(5)(5)=1225=13\begin{vmatrix} 4 & 5 \\ 5 & 3 \end{vmatrix} = (4)(3) - (5)(5) = 12 - 25 = -13

So, det(A)=1(1)2(19)+3(13)=1+3839=0\text{det}(A) = -1 \cdot (-1) - 2 \cdot (-19) + 3 \cdot (-13) = 1 + 38 - 39 = 0.

2. Find the cofactor matrix Cof(A)\text{Cof}(A):

Cof(A)=[(1)1913(21)13515104]=[119132113515104]\text{Cof}(A) = \begin{bmatrix} -(-1) & -19 & -13 \\ -(-21) & -13 & 5 \\ -15 & 10 & 4 \end{bmatrix} = \begin{bmatrix} 1 & -19 & -13 \\ 21 & -13 & 5 \\ -15 & 10 & 4 \end{bmatrix}

3. Find the adjugate matrix adj A\text{adj } A (transpose of Cof(A)\text{Cof}(A)):

adj A=[121151913101354]\text{adj } A = \begin{bmatrix} 1 & 21 & -15 \\ -19 & -13 & 10 \\ -13 & 5 & 4 \end{bmatrix}

4. Compute A(adj A)A (\text{adj } A):

A(adj A)=[123457534][121151913101354]A (\text{adj } A) = \begin{bmatrix}-1 & 2 & 3 \\ 4 & 5 & 7 \\ 5 & 3 & 4 \end{bmatrix} \begin{bmatrix} 1 & 21 & -15 \\ -19 & -13 & 10 \\ -13 & 5 & 4 \end{bmatrix}

Performing the matrix multiplication:

A(adj A)=[(11+2(19)+3(13))(121+2(13)+35)(1(15)+210+34)(41+5(19)+7(13))(421+5(13)+75)(4(15)+510+74)(51+3(19)+4(13))(521+3(13)+45)(5(15)+310+44)]A (\text{adj } A) = \begin{bmatrix} (-1 \cdot 1 + 2 \cdot (-19) + 3 \cdot (-13)) & (-1 \cdot 21 + 2 \cdot (-13) + 3 \cdot 5) & (-1 \cdot (-15) + 2 \cdot 10 + 3 \cdot 4) \\ (4 \cdot 1 + 5 \cdot (-19) + 7 \cdot (-13)) & (4 \cdot 21 + 5 \cdot (-13) + 7 \cdot 5) & (4 \cdot (-15) + 5 \cdot 10 + 7 \cdot 4) \\ (5 \cdot 1 + 3 \cdot (-19) + 4 \cdot (-13)) & (5 \cdot 21 + 3 \cdot (-13) + 4 \cdot 5) & (5 \cdot (-15) + 3 \cdot 10 + 4 \cdot 4) \end{bmatrix}

Calculating each element:

A(adj A)=[000000000]A (\text{adj } A) = \begin{bmatrix} 0 & 0 & 0 \\ 0 & 0 & 0 \\ 0 & 0 & 0 \end{bmatrix}

Therefore, A(adj A)=0A (\text{adj } A) = 0.

3. (b) Solve the inequality4x5212\frac{4x - 5}{2} \leq 12 and graph the solution set.

Solution :

To solve the inequality 4x5212\frac{4x - 5}{2} \leq 12, follow these steps:

  1. First, multiply both sides of the inequality by 2 to eliminate the fraction:
  2. 4x52×212×2\frac{4x - 5}{2} \times 2 \leq 12 \times 2
    4x5244x - 5 \leq 24
  3. Next, add 5 to both sides of the inequality to isolate the term with xx:
  4. 4x5+524+54x - 5 + 5 \leq 24 + 5
    4x294x \leq 29
  5. Finally, divide both sides by 4 to solve for xx:
  6. 4x4294\frac{4x}{4} \leq \frac{29}{4}
    x294x \leq \frac{29}{4}

So, the solution to the inequality is x294x \leq \frac{29}{4}, which can also be written as x7.25x \leq 7.25.

image

3. (c) Solve the equation

8x314x2+7x1=08x^3 - 14x^2 + 7x - 1 = 0, given that roots are in GP.

Solution :

Let the roots be aa, arar, and ar2ar^2 since the roots are in geometric progression (GP).

By Vieta's formulas for the cubic equation ax3+bx2+cx+d=0ax^3 + bx^2 + cx + d = 0:

Sum of the roots:

a+ar+ar2=baa + ar + ar^2 = \frac{-b}{a}
a(1+r+r2)=148a(1 + r + r^2) = \frac{14}{8}
a(1+r+r2)=74a(1 + r + r^2) = \frac{7}{4}

Sum of the product of roots taken two at a time:

aar+aar2+arar2=caa \cdot ar + a \cdot ar^2 + ar \cdot ar^2 = \frac{c}{a}
a2r+a2r2+a2r3=78a^2r + a^2r^2 + a^2r^3 = \frac{7}{8}
a2r(1+r+r2)=78a^2r(1 + r + r^2) = \frac{7}{8}

Product of the roots:

aarar2=daa \cdot ar \cdot ar^2 = \frac{-d}{a}
a3r3=18a^3r^3 = \frac{1}{8}
a3r3=18a^3r^3 = \frac{1}{8}

From this, we get:

a=18r33=12ra = \sqrt[3]{\frac{1}{8r^3}} = \frac{1}{2r}

Substituting a=12ra = \frac{1}{2r} into the sum of the roots equation:

12r(1+r+r2)=74\frac{1}{2r}(1 + r + r^2) = \frac{7}{4}
1+r+r2=7r21 + r + r^2 = \frac{7r}{2}
2+2r+2r2=7r2 + 2r + 2r^2 = 7r
2r25r+2=02r^2 - 5r + 2 = 0

Solving this quadratic equation for rr:

r=5±25164r = \frac{5 \pm \sqrt{25 - 16}}{4}
r=5±34r = \frac{5 \pm 3}{4}
r=2r = 2 or r=12r = \frac{1}{2}

Using rr values in a=12ra = \frac{1}{2r}:

For r=2r = 2:

a=122=14a = \frac{1}{2 \cdot 2} = \frac{1}{4}

The roots are:

a=14,ar=12,ar2=1a = \frac{1}{4}, ar = \frac{1}{2}, ar^2 = 1

For r=12r = \frac{1}{2}:

a=1212=1a = \frac{1}{2 \cdot \frac{1}{2}} = 1

The roots are:

a=1,ar=12,ar2=14a = 1, ar = \frac{1}{2}, ar^2 = \frac{1}{4}

Therefore, the roots are:

14,12,1\frac{1}{4}, \frac{1}{2}, 1 or 1,12,141, \frac{1}{2}, \frac{1}{4}

3. (d) Verify that f(x)=1+x2ln(1x)f(x) = 1 + x^2 \ln(\frac{1}{x}) has a local maximum at x=1e,(x>0)x = \frac{1}{\sqrt{e}},(x > 0).

Solution :

To verify that f(x)=1+x2ln(1x)f(x) = 1 + x^2 \ln(\frac{1}{x}) has a local maximum at x=1ex = \frac{1}{\sqrt{e}}, we first find the first and second derivatives of f(x)f(x).

First, rewrite the function for convenience:

f(x)=1+x2ln(1x)=1+x2(ln(x))=1x2ln(x)f(x) = 1 + x^2 \ln(\frac{1}{x}) = 1 + x^2 (-\ln(x)) = 1 - x^2 \ln(x)

1. Find the first derivative f(x)f'(x):

f(x)=ddx(1x2ln(x))f'(x) = \frac{d}{dx} \left( 1 - x^2 \ln(x) \right)

Using the product rule for differentiation, we get:

f(x)=2xln(x)x21x=2xln(x)xf'(x) = -2x \ln(x) - x^2 \cdot \frac{1}{x} = -2x \ln(x) - x

So, the first derivative is:

f(x)=2xln(x)xf'(x) = -2x \ln(x) - x

2. Set the first derivative equal to zero to find critical points:

2xln(x)x=0-2x \ln(x) - x = 0

Factor out x-x:

x(2ln(x)+1)=0-x (2 \ln(x) + 1) = 0

So, we have two solutions:

x=0x = 0 (not in the domain since x>0x > 0) and 2ln(x)+1=02 \ln(x) + 1 = 0

Solve for xx:

2ln(x)=12 \ln(x) = -1

ln(x)=12\ln(x) = -\frac{1}{2}

x=e12=1ex = e^{-\frac{1}{2}} = \frac{1}{\sqrt{e}}

3. Verify the second derivative to confirm if it is a local maximum:

Find the second derivative f(x)f''(x):

f(x)=ddx(2xln(x)x)f''(x) = \frac{d}{dx} \left( -2x \ln(x) - x \right)

Using the product rule and the chain rule, we get:

f(x)=2(ln(x)+1)1f''(x) = -2 (\ln(x) + 1) - 1

Simplify:

f(x)=2ln(x)21=2ln(x)3f''(x) = -2 \ln(x) - 2 - 1 = -2 \ln(x) - 3

4. Evaluate the second derivative at x=1ex = \frac{1}{\sqrt{e}}:

f(1e)=2ln(1e)3f''\left( \frac{1}{\sqrt{e}} \right) = -2 \ln\left( \frac{1}{\sqrt{e}} \right) - 3

Since ln(1e)=12\ln\left( \frac{1}{\sqrt{e}} \right) = -\frac{1}{2}, we have:

f(1e)=2(12)3=13=2f''\left( \frac{1}{\sqrt{e}} \right) = -2 \left( -\frac{1}{2} \right) - 3 = 1 - 3 = -2

Since f(1e)<0f''\left( \frac{1}{\sqrt{e}} \right) < 0, the function has a local maximum at x=1ex = \frac{1}{\sqrt{e}}.

4. (a) Evaluate: limx01+2x12xx\lim_{{x \to 0}} \frac{\sqrt{1 + 2x} - \sqrt{1 - 2x}}{x}.

Solution :

To evaluate the limitlimx01+2x12xx\lim_{{x \to 0}} \frac{\sqrt{1 + 2x} - \sqrt{1 - 2x}}{x}, we can use the technique of multiplying by the conjugate to simplify the expression.

Multiply the numerator and the denominator by the conjugate of the numerator:

1+2x12xx1+2x+12x1+2x+12x\frac{\sqrt{1 + 2x} - \sqrt{1 - 2x}}{x} \cdot \frac{\sqrt{1 + 2x} + \sqrt{1 - 2x}}{\sqrt{1 + 2x} + \sqrt{1 - 2x}}

This gives:

(1+2x12x)(1+2x+12x)x(1+2x+12x)\frac{(\sqrt{1 + 2x} - \sqrt{1 - 2x})(\sqrt{1 + 2x} + \sqrt{1 - 2x})}{x (\sqrt{1 + 2x} + \sqrt{1 - 2x})}

The numerator simplifies to:

(1+2x)2(12x)2=(1+2x)(12x)=1+2x1+2x=4x(\sqrt{1 + 2x})^2 - (\sqrt{1 - 2x})^2 = (1 + 2x) - (1 - 2x) = 1 + 2x - 1 + 2x = 4x

So the expression becomes:

4xx(1+2x+12x)\frac{4x}{x (\sqrt{1 + 2x} + \sqrt{1 - 2x})}

We can cancel out the xx in the numerator and denominator:

41+2x+12x\frac{4}{\sqrt{1 + 2x} + \sqrt{1 - 2x}}

Now, take the limit as x0x \to 0:

limx041+2x+12x\lim_{{x \to 0}} \frac{4}{\sqrt{1 + 2x} + \sqrt{1 - 2x}}

When x=0x = 0, the expression simplifies to:

41+0+10=41+1=41+1=42=2\frac{4}{\sqrt{1 + 0} + \sqrt{1 - 0}} = \frac{4}{\sqrt{1} + \sqrt{1}} = \frac{4}{1 + 1} = \frac{4}{2} = 2

Therefore, the limit is:

22

4. (b) Find the shortest distance between the lines:r1=(3i^+4j^2k^)+t(i^+2j^+k^)\vec{r_1} = (3\hat{i} + 4\hat{j} - 2\hat{k}) + t( - \hat{i} + 2\hat{j} + \hat{k})andr2=(7i^+2j^+k^)+t(3i^+2j^3k^)\vec{r_2} = ( - 7\hat{i} + 2\hat{j} + \hat{k}) + t(3\hat{i} + 2\hat{j} - 3\hat{k}).

Solution :

To find the shortest distance between two skew lines, we first determine a vector between a point on each line. Let's denote these vectors as:

a=3i^+4j^2k^\vec{a} = 3\hat{i} + 4\hat{j} - 2\hat{k} (from r1\vec{r_1})

b=7i^+2j^+k^\vec{b} = -7\hat{i} + 2\hat{j} + \hat{k} (from r2\vec{r_2})

Next, we find the direction vectors of both lines:

d1=i^+2j^+k^\vec{d_1} = -\hat{i} + 2\hat{j} + \hat{k} (direction vector of r1\vec{r_1})

d2=3i^+2j^3k^\vec{d_2} = 3\hat{i} + 2\hat{j} - 3\hat{k} (direction vector of r2\vec{r_2})

Now, calculate the cross product of d1\vec{d_1} and d2\vec{d_2} to find a vector perpendicular to both direction vectors:

d1×d2=i^j^k^121323\vec{d_1} \times \vec{d_2} = \begin{vmatrix} \hat{i} & \hat{j} & \hat{k} \\ -1 & 2 & 1 \\ 3 & 2 & -3 \end{vmatrix}

Compute the determinant:

d1×d2=i^(2(3)1(2))j^((1)(3)3(1))+k^((1)(2)3(2))\vec{d_1} \times \vec{d_2} = \hat{i}(2(-3) - 1(2)) - \hat{j}((-1)(-3) - 3(1)) + \hat{k}((-1)(2) - 3(2))

d1×d2=i^(62)j^(33)+k^(26)\vec{d_1} \times \vec{d_2} = \hat{i}(-6 - 2) - \hat{j}(3 - 3) + \hat{k}(-2 - 6)

d1×d2=i^(8)j^(0)+k^(8)\vec{d_1} \times \vec{d_2} = \hat{i}(-8) - \hat{j}(0) + \hat{k}(-8)

d1×d2=8i^8k^\vec{d_1} \times \vec{d_2} = -8\hat{i} - 8\hat{k}

Now, find the vector between a point on each line:

AB=ab=(3i^+4j^2k^)(7i^+2j^+k^)\vec{AB} = \vec{a} - \vec{b} = (3\hat{i} + 4\hat{j} - 2\hat{k}) - (-7\hat{i} + 2\hat{j} + \hat{k})

AB=10i^+2j^3k^\vec{AB} = 10\hat{i} + 2\hat{j} - 3\hat{k}

Calculate the magnitude of d1×d2\vec{d_1} \times \vec{d_2}:

d1×d2=(8)2+02+(8)2=64+64=128=82\left| \vec{d_1} \times \vec{d_2} \right| = \sqrt{(-8)^2 + 0^2 + (-8)^2} = \sqrt{64 + 64} = \sqrt{128} = 8\sqrt{2}

Calculate the shortest distance dd between the lines using the formula:

d=AB(d1×d2)d1×d2d = \frac{\left| \vec{AB} \cdot (\vec{d_1} \times \vec{d_2}) \right|}{\left| \vec{d_1} \times \vec{d_2} \right|}

Calculate AB(d1×d2)\vec{AB} \cdot (\vec{d_1} \times \vec{d_2}):

AB(d1×d2)=(10i^+2j^3k^)(8i^8k^)\vec{AB} \cdot (\vec{d_1} \times \vec{d_2}) = (10\hat{i} + 2\hat{j} - 3\hat{k}) \cdot (-8\hat{i} - 8\hat{k})

AB(d1×d2)=10(8)+2(0)3(8)\vec{AB} \cdot (\vec{d_1} \times \vec{d_2}) = 10(-8) + 2(0) - 3(-8)

AB(d1×d2)=80+24=56\vec{AB} \cdot (\vec{d_1} \times \vec{d_2}) = -80 + 24 = -56

Therefore, the shortest distance dd between the lines is:

d=5682=5682=5681.4145611.3124.95d = \frac{|-56|}{8\sqrt{2}} = \frac{56}{8\sqrt{2}} = \frac{56}{8 \cdot 1.414} \approx \frac{56}{11.312} \approx 4.95

So, the shortest distance between the lines is approximately 4.954.95 units.

4. (c) Determine the length of curvey=23x32y = \frac{2}{3}x^{\frac{3}{2}}from (0,0)(0, 0) to (1,23)(1, \frac{2}{3})

Solution :

4. (d) Find the sum of all the integers between 100 and 1000 that are divisible by 7.

Solution :

To find the sum of all integers between 100 and 1000 that are divisible by 7, we first determine the range of integers and then apply the formula for the sum of an arithmetic sequence.

Identify the first and last integers in the range:

First integer divisible by 7 in the range: 1007×7=15×7=105\lceil \frac{100}{7} \rceil \times 7 = 15 \times 7 = 105

Last integer divisible by 7 in the range: 10007×7=142×7=994\lfloor \frac{1000}{7} \rfloor \times 7 = 142 \times 7 = 994

These integers form an arithmetic sequence where:

  • First term a=105a = 105
  • Last term l=994l = 994
  • Common difference d=7d = 7

Number of terms nn in the sequence:

n=lad+1=9941057+1=8897+1=127+1=128n = \frac{l - a}{d} + 1 = \frac{994 - 105}{7} + 1 = \frac{889}{7} + 1 = 127 + 1 = 128

Sum of the arithmetic sequence SnS_n:

Sn=n2×(a+l)S_n = \frac{n}{2} \times (a + l)

Calculate the sum SnS_n:

Sn=1282×(105+994)=64×1099=70136S_n = \frac{128}{2} \times (105 + 994) = 64 \times 1099 = 70136

Therefore, the sum of all integers between 100 and 1000 that are divisible by 7 is 70136\boxed{70136}.

5. (a) Determine the area between the two curves y=3+2xy = 3 + 2x, y=3xy = 3 - x, 0x30 \leq x \leq 3 using integration. (5 marks)

Answer :

To find the area between the curves y=3+2xy = 3 + 2x and y=3xy = 3 - x over the interval 0x30 \leq x \leq 3, we first find the points of intersection by setting the equations equal to each other:

3+2x=3x3 + 2x = 3 - x

Simplifying, we get:

2x+x=02x + x = 0

3x=03x = 0

x=0x = 0

Since this point is within the given interval 0x30 \leq x \leq 3, we verify the second point at x=3x = 3.

At x=3x = 3:

y=3+2(3)=3+6=9y = 3 + 2(3) = 3 + 6 = 9

y=33=0y = 3 - 3 = 0

Thus, the curves do not intersect again in the interval, so we proceed with integration over the entire interval:

The area between the curves from x=0x = 0 to x=3x = 3 is given by:

03[(3+2x)(3x)]dx\int_{0}^{3} [(3 + 2x) - (3 - x)] \, dx

Simplifying the integrand:

(3+2x)(3x)=3+2x3+x=3x(3 + 2x) - (3 - x) = 3 + 2x - 3 + x = 3x

So, the integral becomes:

033xdx\int_{0}^{3} 3x \, dx

We integrate 3x3x:

3xdx=3x22\int 3x \, dx = \frac{3x^2}{2}

Evaluating this from 00 to 33:

[3x22]03=3(3)223(0)22\left[ \frac{3x^2}{2} \right]_{0}^{3} = \frac{3(3)^2}{2} - \frac{3(0)^2}{2}

This simplifies to:

3920=272\frac{3 \cdot 9}{2} - 0 = \frac{27}{2}

Thus, the area between the curves is 272\frac{27}{2} square units.

5. (b) Find the direction cosines of the lines passing through the two points (1, 2, 3) and (-1, 1, 0).

Answer :

To find the direction cosines of the line passing through the points (1, 2, 3) and (-1, 1, 0), we use the formula:

If r1=(x1,y1,z1)\vec{r_1} = (x_1, y_1, z_1) and r2=(x2,y2,z2)\vec{r_2} = (x_2, y_2, z_2) are two points, then the direction vector d\vec{d} is given by:

d=(x2x1,y2y1,z2z1)\vec{d} = (x_2 - x_1, y_2 - y_1, z_2 - z_1)

Now, calculate the direction vector d\vec{d} from the given points:

d=(11,12,03)=(2,1,3)\vec{d} = (-1 - 1, 1 - 2, 0 - 3) = (-2, -1, -3)

Next, calculate the magnitude of the direction vector d\vec{d}:

d=(2)2+(1)2+(3)2=4+1+9=14\left| \vec{d} \right| = \sqrt{(-2)^2 + (-1)^2 + (-3)^2} = \sqrt{4 + 1 + 9} = \sqrt{14}

Now, find the direction cosines l,m,nl, m, n:

l=214,m=114,n=314l = \frac{-2}{\sqrt{14}}, \quad m = \frac{-1}{\sqrt{14}}, \quad n = \frac{-3}{\sqrt{14}}

Therefore, the direction cosines of the line passing through the points (1, 2, 3) and (-1, 1, 0) are (214,114,314)\left( \frac{-2}{\sqrt{14}}, \frac{-1}{\sqrt{14}}, \frac{-3}{\sqrt{14}} \right).

5. (c) Find the maximum value of 2a+5b2a + 5b subject to the following constraints:
3a2b6-3a - 2b \leq -6
2a+b2-2a + b \leq 2
4a+6b244a + 6b \leq 24
2a3b32a - 3b \leq 3
a0a \geq 0 and b0b \geq 0. (5 marks)

Answer :

To find the maximum value of 2a+5b2a + 5b subject to the given constraints, we will solve this linear programming problem graphically.

The constraints are:

3a2b6-3a - 2b \leq -6
2a+b2-2a + b \leq 2
4a+6b244a + 6b \leq 24
2a3b32a - 3b \leq 3
a0a \geq 0
b0b \geq 0

First, we rewrite the constraints in a more convenient form:

3a+2b63a + 2b \geq 6
2ab22a - b \geq -2
4a+6b244a + 6b \leq 24
2a3b32a - 3b \leq 3
a0a \geq 0
b0b \geq 0

Next, we find the intersection points of the boundary lines to determine the feasible region:

  1. For 3a+2b=63a + 2b = 6 and a0a \geq 0, b0b \geq 0, we get the intercepts:
    • When a=0a = 0, b=3b = 3
    • When b=0b = 0, a=2a = 2
  2. For 2ab=22a - b = -2 and a0a \geq 0, b0b \geq 0, we get the intercepts:
    • When a=0a = 0, b=2b = -2 (but since b0b \geq 0, this point is not in the feasible region)
    • When b=0b = 0, a=1a = -1 (but since a0a \geq 0, this point is not in the feasible region)
  3. For 4a+6b=244a + 6b = 24 and a0a \geq 0, b0b \geq 0, we get the intercepts:
    • When a=0a = 0, b=4b = 4
    • When b=0b = 0, a=6a = 6
  4. For 2a3b=32a - 3b = 3 and a0a \geq 0, b0b \geq 0, we get the intercepts:
    • When a=0a = 0, b=1b = -1 (but since b0b \geq 0, this point is not in the feasible region)
    • When b=0b = 0, a=1.5a = 1.5

The vertices of the feasible region are:

(0,0),(2,0),(0,4),(185,245),(2413,2113)\left(0, 0\right), \left(2, 0\right), \left(0, 4\right), \left( \frac{18}{5}, \frac{24}{5} \right), \left( \frac{24}{13}, \frac{21}{13} \right)

Finally, we evaluate the objective function 2a+5b2a + 5b at each vertex:

  • At (0,0)\left(0, 0\right): 2(0)+5(0)=02(0) + 5(0) = 0
  • At (2,0)\left(2, 0\right): 2(2)+5(0)=42(2) + 5(0) = 4
  • At (0,4)\left(0, 4\right): 2(0)+5(4)=202(0) + 5(4) = 20
  • At (185,245)\left( \frac{18}{5}, \frac{24}{5} \right): 2(185)+5(245)=365+1205=1565=31.22\left(\frac{18}{5}\right) + 5\left(\frac{24}{5}\right) = \frac{36}{5} + \frac{120}{5} = \frac{156}{5} = 31.2
  • At (2413,2113)\left( \frac{24}{13}, \frac{21}{13} \right): 2(2413)+5(2113)=4813+10513=1531311.772\left(\frac{24}{13}\right) + 5\left(\frac{21}{13}\right) = \frac{48}{13} + \frac{105}{13} = \frac{153}{13} \approx 11.77

The maximum value of 2a+5b2a + 5b is 31.231.2 at the vertex (185,245)\left( \frac{18}{5}, \frac{24}{5} \right).

5. (d) Reduce the matrix [538011110]\begin{bmatrix} 5 & 3 & 8 \\ 0 & 1 & 1 \\ 1 & -1 & 0 \end{bmatrix} to normal form and hence find its rank. (5 marks)

Answer :

To reduce the matrix to its normal form and find its rank, we perform row operations:

Given matrix:

[538011110]\begin{bmatrix} 5 & 3 & 8 \\ 0 & 1 & 1 \\ 1 & -1 & 0 \end{bmatrix}

Step-by-step reduction:

  1. Subtract 5 times the second row from the first row (R1 → R1 - 5R2):
  2. [503011110]\begin{bmatrix} 5 & 0 & 3 \\ 0 & 1 & 1 \\ 1 & -1 & 0 \end{bmatrix}
  3. Swap the first and third rows to bring a non-zero entry to the pivot position:
  4. [110011503]\begin{bmatrix} 1 & -1 & 0 \\ 0 & 1 & 1 \\ 5 & 0 & 3 \end{bmatrix}
  5. Subtract 5 times the first row from the third row (R3 → R3 - 5R1):
  6. [110011053]\begin{bmatrix} 1 & -1 & 0 \\ 0 & 1 & 1 \\ 0 & 5 & 3 \end{bmatrix}
  7. Subtract 5 times the second row from the third row (R3 → R3 - 5R2):
  8. [110011002]\begin{bmatrix} 1 & -1 & 0 \\ 0 & 1 & 1 \\ 0 & 0 & -2 \end{bmatrix}

Now, the matrix is in row echelon form (REF). To find the rank, count the number of non-zero rows:

Since there are 3 non-zero rows, the rank of the matrix is 33.

Therefore, the reduced row echelon form (RREF) of the matrix is [110011002]\begin{bmatrix} 1 & -1 & 0 \\ 0 & 1 & 1 \\ 0 & 0 & -2 \end{bmatrix} and its rank is 33.